Categories
Exam Questions Harvard Statistics

Harvard. Enrollment and Exam for Statistics. Ripley, 1902-1903

At the turn of the 20th century the breadth of Harvard’s course offerings grew faster than the depth of its instructional bench. Thomas Nixon Carver was teaching economic theory, sociology, social reform, and agricultural economics while William Zebina Ripley covered statistics, railroads, European resources, industrial and labor organization.

This post adds another year’s worth of Harvard statistics exams to the Economics in the Rear-View Mirror collection of transcribed artifacts. 

______________________________

Course Description

  1. hf. Statistics. — Theory, method, and practice. Half-course. Tu., at 11. Professor Ripley.

This course is intended to serve rather as an analysis of methods of research and sources of information than as a description of mere results. A brief history of statistics will be followed by an account of modes of collecting and tabulating census and other statistical material in the United States and abroad, the scientific use and interpretation of results by the mean, the average, seriation, the theory of probability, etc. The main divisions of vital statistics, relating to birth, marriage, morbidity, and mortality, life tables, etc.; the statistics of trade and commerce, such as price indexes, etc.; industrial statistics relating to labor, wages, and employment; statistics of agriculture, manufactures, and transportation, will be them considered in order. The principal methods of graphic representation will be comprehended, and laboratory work, amounting to not less than two hours per week, in the preparation of charts, maps, and diagrams from original material, will be required.

Course 4 is open to students who have taken Economies 1; and it is also open to Juniors and Seniors who are taking Economics 1. It is especially recommended, in connection with Economies 2, for all candidates for advanced degrees.

Source: Harvard University. Faculty of Arts and Sciences, Division of History and Political Science  [Comprising the Departments of History and Government and Economics], 1902-03. Published in The University Publications, New Series, no. 55. June 14, 1902.

______________________________

Course Enrollment

Economics 4. hf. Professor Ripley. — Statistics. Theory, method, and practice.

Total 15: 10 Gr., 2 Se., 2 Ju., 1 Other.

Source: Harvard University. Annual Report of the President of Harvard College, 1902-03, p. 68.

______________________________

Economics 4
Mid-Year Examination
1902-03

  1. The population of Massachusetts in 1870 was 1,457,351, in 1875 was 1,651,912, in 1880 was 1,783,085. The births in 1878 were 41,238. The Massachusetts Registration Report gives the birth rate in that year as 24.73. How was this obtained?
  2. The birth rates for 1885 in Massachusetts by counties were as follows: Barnstable, 17.4; Berkshire, 25.7; Dukes and Nantucket, 14; Suffolk, 28.7; Middlesex, 24.7; Worcester city, 27.1; Fall River city, 31.07; Cambridge, 27.04; Newburyport, 17.79, etc.
    How would you determine the real significance of these differences?
  3. Padding of the enumerator’s returns in Delaware counties in 1900, being known, how could you estimate the probable amount, assuming no considerable migration of population to have occurred?
  4. What are the main tests for determining the amount of migration in a given population between census periods?
  5. What single merit has the card system of tabulation used by the Massachusetts Bureau of Labor Statistics, over the use of electric machines as in the Federal Census?
  6. Compare the rate of increase of the population of the United States with that of principal European countries. What probable future movement is indicated?
  7. What are the principal demographic results of an inequality in the distribution of the sexes? Illustrate by the United States.

Source: Harvard University Archives. Mid-year Examinations 1852-1943. Box 6. Papers (in the bound volume Examination Papers Mid-years 1902-1903).

______________________________

Economics 4
Year-End Examination
1902-03

  1. What is meant by the “duration of life”? What figures are apt to be confused with it in mortality statistics?
  2. What are the main objections to the use of index numbers, illustrating by examples?
  3. What is the “modulus” as applied to wage statistics? In what different ways may it be ascertained?
  4. Where would you seek for examples of the best practice in interpretation of (a) price statistics; (b) wage statistics?
  5. May the “cost of production” in manufactures be determined with precision? Where has the attempt has been made and with what results?
  6. Where are the best statistics of imports and exports compiled? How does the system differ from those of the United States?
  7. How is a logarithmic curve constructed; and for what purpose?

Source: Harvard University Archives. Examination Papers 1873-1915. Box 6. Papers Set for Final Examinations in History, Government, Economics, History of Religions, Philosophy, Education, Fine Arts, Architecture, Landscape Architecture, Music in Harvard College, June 1903 (in the bound volume Examination Papers 1902-1903).

Image Source: MIT Museum website. William Zebina Ripley. Image colorized by Economics in the Rear-View Mirror.

Categories
Exam Questions Johns Hopkins Suggested Reading Syllabus Undergraduate

John Hopkins. Economic Fluctuations and Fiscal Policy. Course outline, reading list, exams. Domar, 1956

Evsey Domar turned 42 years old towards the end of the Spring term of 1955-56 when he taught his intermediate fiscal policy course to Johns Hopkins’ undergraduates. From his papers at Duke’s Economists’ Papers Archive we can bring together the tightly focussed reading list, two midterm exams, and the final exam for Political Economy 4. 

One notes that the actual dates of the mid-term exams were lagged one week relative to the announced dates in the syllabus. Happens to the best of us. I wonder if students still (ever?) read the syllabus back in the middle of the 20th century. 

________________________

Course Announcement

Political Economy
Specialized intermediate work

Economic Fluctuations and Fiscal Policy 4. Professor Domar. Three hours weekly, second term.

The nature and causes of economic fluctuations. The economic role of government. Principal policy measures designed to achieve economic stability.

Prerequisite: Political Economy 3, or its equivalent.

Source: Johns Hopkins University. Undergraduate Programs, Announcements of Courses 1955-1956 in Circular 1955-1956. New Series 1955, Number 8, p. 102.

________________________

Course Outline and Readings

THE JOHNS HOPKINS UNIVERSITY
ECONOMIC FLUCTUATIONS
AND FISCAL POLICY

(Political Economy 4)

E. D. Domar
Spring Term 1955-56

Course Schedule

SOURCES:

On College Reserve:

Colm, Gerhard, Essays in Public Finance and Fiscal Policy, Oxford University Press, New York 1955.

Due, John F., Government Finance—an Economic Analysis, Richard D. Irwin, Inc., Homewood, Ill., 1954.

Gordon, Robert A., Business Fluctuations, Harper & Brothers, New York, 1952.

Lindholm, Richard W., J. J. Balles, J. M. Hunter, Principles of Money and Banking Related to National Income and Fiscal Policy,W. W. Norton & Co., New York, 1954.

Public Finance and Full Employment, published by the Board of Governors of the Federal Reserve System, Washington, 1945.

Ritter, Lawrence S., Money and Economic Activity, Houghton Mifflin Co., Boston, 1952.

To Be Acquired by the Students:

Maxwell, James A., Fiscal Policy, Henry Holt & Co., New York, 1955.

Economic Report of the President, January 1956.

SCHEDULE:

Week of February 13th:

Maxwell, Ch. 1,
Ritter, pp. 20-36
Lindholm, pp. 17-31.

Week of February 20th:

Ritter, pp. 99-113,
Maxwell, Ch. 2.

Week of February 27th:

Ritter, pp. 120-130,
Lindholm, pp. 330-348

Week of March 5th:

Lindholm, pp. 370-408.

HOUR EXAMINATION: March 12th

Week of March 12th:

Maxwell, Ch. 3, 4 & 5.

Week of March 19th:

Maxwell, Ch. 6, 7, & 8,
Federal Reserve, pp. 1-21,
Colm, pp. 188-219.

Week of March 26th:

Maxwell, Ch. 9, 10, & 11,
Colm, pp. 258-286.

Week of April 2nd:

Maxwell, Ch. 12 & 13,
Federal Reserve, pp. 22-52,
Review – Due, pp. 29-61, 427-39.

Week of April 9th:

Maxwell, Ch. 14 & 15.

HOUR EXAMINATION: April 16th

Week of April 16th:

Federal Reserve, pp. 53-68, 101-130.

Week of April 23rd:

Review – Gordon, Ch. 13 & 14, and pp. 559-74.

Week of April 30th:

Gordon, Ch. 16, 17 & 18.

Week of May 7th:

Economic Report of the President

Week of May 14th:

Economic Report of the President

Week of 21st:

General Review of the Course

Source: Duke University. David M. Rubenstein Rare Book and Manuscript Library. Economists’ Papers Archive. Papers of Evsey Domar. Box 15, Folder “MacroEconomics, Old Reading Lists”.

________________________

First Hour Test

THE JOHNS HOPKINS UNIVERSITY
Economic Fluctuations and Fiscal Policy
(Political Economy 4)
Spring Term 1955-56

March 19, 1956

E.D. Domar

Answer all questions in any order you wish. Indicate carefully every step in your reasoning.

  1. (40%) Write a comprehensive essay on the subject of “Central Bank Monetary Policy” with special reference to our Federal Reserve System. Your essay should include the following points:
    1. The structure of the Federal Reserve System.
    2. The relation between commercial and Federal Reserve Banks.
    3. Objectives of Federal Reserve Policy.
    4. Powers given to the Federal Reserve System and methods used by it to achieve the objectives indicated in (3) under different economic conditions.
      1. General measures
      2. Selective measures
    5. Evaluate the performance of the Federal Reserve System since its inception.
      How successful has it been in achieving the objectives stated in (3)?
    6. Conclusion: the virtues and defects of Monetary Policy.
  2. (25%) Indicate clearly how DEMAND DEPOSITS, REQUIRED RESERVES, EXISTING RESERVES and EXCESS RESERVES of the commercial member banks taken as a whole are affected by the following transactions.
    Assume that all payments are made by check, that the member banks add all receipts to, and subtract all amounts paid out from, their reserves with the Federal Reserve Banks, and that the U.S. Treasury keeps all its funds with the Federal Reserve Banks:
  3. 25% Legal requirements are 15 per cent.
  4. When a transaction consists of several parts, indicate each part separately and then show the total effect.
    AFTER EACH TRANSACTION GIVE A BRIEF VERBAL ANALYSIS OF ITS ECONOMIC EFFECTS.

    1. The U.S. Treasury collects $15 million of corporate income taxes from the U.S. Steel Corporation and uses the proceeds to redeem a bond held by Mr. Smith who deposits the check with his bank.
    2. Same as (1), but the bond is held by the First National Bank.
    3. Jones borrows $1000 from the First National Bank. After a while he uses the proceeds to meet his payroll. His employees invest their earnings in Federal bonds.
    4. The U.S. Treasury sells bonds for $100 million to the public, and uses the proceeds to buy land for highway construction. The owners of the land deposit their checks at their banks. The Federal Reserve Banks buy $100 million worth of Federal bonds from (a) the public, and (b) commercial banks.
    5. The Federal Reserve Board changes reserve requirements from 20 to 18 per cent. (Assume that the amount of deposits outstanding equals to $100 billion.) Thereupon banks extend loans to their customers of $1 billion.
  5. (35%) Write a comprehensive essay on the subject of “The Identity and Divergence between Private and Social Cost.” Illustrate your discussion with examples. Why is this question important to the subject matter of our course and to economic policy in general. (No credit will be given for vague generalities.)

Source: Duke University. David M. Rubenstein Rare Book and Manuscript Library. Economists’ Papers Archive. Papers of Evsey Domar, Box 16, Folder “Misc. Examinations”.

________________________

Second Hour Exam

THE JOHNS HOPKINS UNIVERSITY
ECONOMIC FLUCTUATIONS
AND FISCAL POLICY

(Political Economy 4)
Spring Term 1955-56

Hour Examination
April 23, 1956

E.D. Domar

Answer all questions in any order you wish. Indicate carefully every step in your reasoning. No credit will be given for vague generalities.

  1. (15%) Define and describe the following terms or expressions and indicate their use in economic discussions:
    1. The Multiplier;
    2. Parity;
    3. Balanced budget theorem;
    4. Cash vs. conventional budget;
    5. Carryovers and carrybacks;
    6. Income elasticity of taxation;
    7. Regressive taxation;
    8. Payroll taxes;
    9. Grants-in-aid;
    10. Accelerated depreciation.
  1. (20%) Write a comprehensive essay on the subject of “Built-in Flexibility as an Instrument of Fiscal Policy.” Explain what is meant by this expression, how this instrument works, how effective it is likely to be, and what can be done to increase its effectiveness. Give a critical evaluation. Be as comprehensive and specific as you can.
  2. (20%) Write a comprehensive essay on the subject of “The Agricultural Problem in the United States since the Second World War.” Explain the origin and causes of the problem, government policies which have been adopted, and their effectiveness in dealing with the problem. Indicate and justify your own recommendations.
  3. (30%) Analyze with great care all important economic effects of agricultural price support program on the assumption of (1) that the funds for this purpose are raised by borrowing, and (2) that they are raised by taxation, in both cases under conditions of (a) unemployment, and (b) full employment. Indicate in all cases what kind of borrowing and what kind of taxation you have in mind. Give examples. When would you recommend one or the other method?
  4. (15%) “The main objective of the Federal policy should be not the balancing of the Federal budget, but of the national economic budget.” Comment.

Source: Duke University. David M. Rubenstein Rare Book and Manuscript Library. Economists’ Papers Archive. Papers of Evsey Domar, Box 16, Folder “Misc. Examinations”.

________________________

Final Exam

THE JOHNS HOPKINS UNIVERSITY
ECONOMIC FLUCTUATIONS
AND FISCAL POLICY

(Political Economy 4)

FINAL EXAMINATION – Three hours
June 1, 1956

E. D. Domar

Answer all questions. Be specific.

  1. (25%) Compare and contrast monetary and fiscal policies as methods of achieving economic stabilization (reasonably full employment without inflation) in a growing society. Include (but don’t limit yourself to) the following points:
    1. The theoretical foundation of each;
    2. Methods used by each;
    3. Effects on distribution of income and wealth;
    4. Social and political effects;
    5. Their effectiveness and limitations.

Do they overlap? Can you work out a synthesis of both?

  1. (10%) Describe how business fluctuations spread internationally and discuss critically the various measures for insuring international stability that have been suggested.
  2. (15%) Suppose that sizable gold deposits were discovered in this country (a) in 1933 and (b) in 1955. Trace the economic effects of the mining of this gold as completely as you can, both on the American economy and on that of other countries.
  3. (20%) Describe the origin, functions and performance of the Council of Economic Advisers from its beginning.
    State and evaluate the basic economic philosophy and the major recommendations of the 1956 Economic Report of the President.
  4. (15%) “One of the first objectives of this Administration should be at least a partial repayment of the Federal Debt. To do otherwise is to undermine the integrity on which this Administration is founded, and to adopt a course which inevitably loads to higher taxes, inflation, the destruction of our national wealth and economic insolvency.” Comment fully.
  5. (15%) Discuss SAVING as an economic problem.

Source: Duke University. David M. Rubenstein Rare Book and Manuscript Library. Economists’ Papers Archive. Papers of Evsey Domar, Box 16, Folder “Misc. Examinations”.

Image source: Duke University. David M. Rubenstein Rare Book and Manuscript Library. Economists’ Papers Archive. Papers of Evsey Domar, Box 18, Folder “Photographs Domar”. Copy also available at the MIT Museum website. Colorized by Economics in the Rear-view Mirror.

Categories
Bibliography Chicago Economists Gender Social Work

Chicago. Economics Ph.D. alumna. Linked publications list. Edith Abbott, 1905

The second woman to receive a Political Economy Ph.D. from the University of Chicago, Edith Abbott, became the first woman dean of a U.S. graduate school in 1924 (The University of Chicago’s Graduate School of Social Service Administration). In her day there were two main paths to an academic career for women economists: home economics and “social economy”. She and her long-time collaborator, Sophonisba Breckinridge, (the first woman to receive a political science Ph.D. at the University of Chicago…note: on an economics topic “A Study of Legal Tender in England“) were fellow research directors at Jane Addams’ Hull House. 

In Germany Sozialpolitik was, like virtually all academic disciplines, Männersache. In Anglo-American academic life social policy was where women could participate.

With this post Economics in the Rear-View Mirror very proudly offers historians of economics and social policy links to well over 90% of her publications.

_________________________________

Life and Career of Edith Abbott
Economics Ph.D. alumna (1905),
Department of Political Economy

University of Chicago

Thesis Title: A statistical study of the wages of unskilled labor in the United States, 1830-1900.
Published in The Journal of Political Economy (June, 1905) as “The Wages of Unskilled Labor in the United States 1850-1900”.

1876, September 26. Born in Grand Island, Nebraska.
1888-93. Graduated as valedictorian of Brownell Hall, a private school in Omaha, Nebraska.
1893-95. Taught at Grand Island High School, Nebraska.
1901. A.B. University of Nebraska.
1901-03. Graduate Student, University of Nebraska. and Instructor in Mathematics, Lincoln High School.
1902. Summer school at the University of Chicago.
1903-05.Fellow, Department of Political Economy, University of Chicago. Supported by J. L. Laughlin and Thorstein Veblen.
1905. Ph.D. in Political Economy from the University of Chicago.
1905Post-Ph.D. she worked two jobs in Boston: (1) Secretary at the Women’s Trade Union League and (2) assisted in the U.S. industrial history research project of Carroll D. Wright for the American Economic Association funded by the Carnegie Institution. She lived at the social settlement Dennison House.
1906. Full-time work for the Carnegie Institution. Moved in January to New York City for research. Lived at College Settlement. Next moved to Washington, D.C.
1906-07. 
With funds from a competitive fellowship awarded by the Association of Collegiate Alumnae, supplemented by Carnegie Institution funds, she went for postgraduate study at the London School of Economics. She took a course “Methods of Social Investigation” taught by Beatrice Webb [see description of Abbott’s own methods course taught 1909-10 at the University of Chicago below]. Lived at St. Hilda’s Settlement in Bethnal Green.
1907-1908. Instructor of economics at Wellesley College.
1908-20. Resident of Hull House. Associate Director of the Chicago School of Civics and Philanthropy.
1909-10.Special Lecturer in Political Economy, University of Chicago.
1918-19. Vice President of the American Economic Association.
1920. Appointed Associate Professor of Social Economy in the Graduate School of Social Service Administration, University of Chicago, Chicago, Ill.
1924-42Dean of the School of Social Service Administration, University of Chicago.
1926. Established Cook County (Illinois) Bureau of Public Welfare.
1927.Together with Sophonisba P. Breckinridge co-founded Social Service Review.
1929-31.  Chaired the Committee on Crime and the Foreign Born of the Wickersham National Commission on Law Observance and Enforcement
1935. Assisted in drafting the Social Security Act.
1942-1953. Dean Emeritus.
1953. Returned to Grand Island, Nebraska and lived with her brother Arthur.
1957, 28 July. Died in Grand Island, Hall County, Nebraska8

Sources:
Costin, Lela B. 1983. Two Sisters for Social Justice: A Biography of Grace and Edith Abbott. Urbana and Chicago: University of Illinois Press.
Deegan, Mary Jo and Michael R. Hill. 1991. “Edith Abbott (1876-1957).” Pp. 29-36 in Women in Sociology: A Bio-Bibliographical Sourcebook, edited by Mary Jo Deegan. New York: Greenwood Press.

______________________________

Edith Abbott report on her year at L.S.E.
1906-07

Your [Association of Collegiate Alumnae] fellow of last year, Miss Abbott, went in September to the London School of Economics where her principal work was the study of statistical methods, taking both lecture and research work with Mr. Bowley, and also taking advantage of the opportunity of attending other lecture courses both in economic theory and in economic history and in methods of social investigation. She also found helpful work in University College, and in the spring attended some lectures at the School of Sociology. The part of the year that she counted most valuable, however, was the time spent with Miss Collet, Investigator of Women’s Trades for the English Labor Department, who for the past 20 years has been studying various questions connected with the employment of women. In the winter she made an investigation in connection with the “Outer London Inquiry,” and in the summer she had an opportunity of studying the working of the Unemployed Act. A short account of one phase of this “Municipal Employment of Unemployed Women in London” will appear in the current number of the Journal of Political Economy. Her History of the Industrial Employment of Women has made some progress. She will publish in the December number of the Journal of Political Economy, “Women in Manufactures: A Supplementary Note,” and in the January, 1907, she published a paper on “The History of the Employment of Women in Cigar-Making.” She has been appointed instructor at Wellesley College to carry on some of Professor Comane’s work during the latter’s leave of absence.

Source: The Association of Collegiate Alumnae Magazine III.17 (Jan. 1908) pp. 140-141.

______________________________

Faculty Blurb and Course Description,
University of Chicago, 1909-10

Edith Abbott, Ph.D., Special Lecturer in Political Economy.

A.B., University of Nebraska, 1901; Ph.D., University of Chicago, 1905; Fellow in Political Economy, ibid., 1903 -05; Research Work for Carnegie Institution of Washington, 1906; European Fellow of the Association of Collegiate Alumnae and Student at the London School of Economics, 1906 -7; Instructor in Political Economy, Wellesley College, 1907-8; Associate Director Chicago School of Civics and Philanthropy, 1908—; Special Lecturer in Political Economy, University of Chicago, 1909—.

  1. Methods of Social Investigation. A course designed primarily to acquaint students with the purpose, methods, and results of the most important work that has been done in the field of social research. Such investigations as Le Play’s Ouvriers européens, Booth’s Life and Labor of the People of London, Rowntree’s Poverty, women in the printing trades, and the recent Dundee and West Ham inquiries will be studied, as well as some selected reports of Royal Commissions and of the English and American Labor Departments. The application of statistical methods to social problems, the collection and tabulation of data, the use and misuse of averages, index-numbers, and weighting will be treated briefly; and the use and limitations of experiment, the interview, the document, and personal observation will be considered.

Students may supplement this course by practical work in investigation in connection with one of the Inquiries being carried on by the Research Department of the School of Civics and Philanthropy. An additional major’s credit will be given to students who give not less than 12 hours a week to this part of the course. Mj. or 2 Mj. Winter Quarter, 9:30, Dr. Abbott.

Source: University of Chicago. Annual Register,  July, 1908—July, 1909 with Announcements for 1909-1910 (Chicago: July 1908),  pp. 50, 237.

_________________________________

Jane Adams of Hull House:
Introducing Edith Abbott

From Jane Addams’ preface to the pamphlet The Wage-Earning Woman and the State by Edith Abbott and Sophonisba P. Breckinridge published by the Boston Equal Suffrage Association for Good Government (1910)

…Miss Edith Abbott was graduated from the University of Nebraska, and later received the degree of Doctor of Philosophy in Economics and Law from the University of Chicago. She was for two years a Fellow of the University, and studied in Europe for one year at the University of London in the School of Economics. After teaching political economy at Wellesley College for one year, she entered the School of Civics and Philanthropy, where she has been Associate Director for the last five years. She is the author of a very authoritative work entitled « Women in Industry; A Study in American Economic » Her knowledge of the conditions surrounding working women is by no means confined to America. She is in constant correspondence with the people most interested in the conditions of working women in England and the continental countries, and by travel and correspondence has kept herself well informed concerning the legal and industrial changes which affect the lives of women the world over. Both Miss [Sophonisba P.] Breckinridge and Miss Abbott are personally acquainted with hundreds of working women. Miss Abbott has been a resident of Hull House for the last few years, and Miss Breckinridge is in residence each year during her three months’ vacation from teaching at the University. They thus add to their scholarly qualifications a keen and living interest in thousands of working women.

JANE ADDAMS.
Hull House, Chicago.

_________________________________

Back story of  the Graduate School of Social Service Administration

The most recently established of the graduate schools of the University makes its entry somewhat timidly for the first time in the rather jovial surroundings and setting provided by the Cap and Gown. The School deals with almost discordantly sombre themes — pauperism, crime, drunkenness, insanity, and vice. Its laboratories are the mean streets of the West Side, the deteriorated area of “Lower North,” the industrial district to the south along the banks of the Calumet. But the School is older, in its traditions at least, than its debut would indicate.

First established more than twenty years ago as the Institute of Social Science under the auspices of the University of Chicago, University College, the School numbered among its first members of its faculty Professor Graham Taylor of the Chicago Commons, Professor Charles Richmond Henderson of the University, and Miss Julia C. Lathrop of Hull House. In May, 1908, the Institute of Social Science became the Chicago School of Civics and Philanthropy and maintained an independent existence from 1908 to 1920, when it was combined with the philanthropic service division that had been organized under the rapidly expanding School of Commerce and Administration, with Mr. [Leon Carroll] Marshall as the first dean of the new School. The present Graduate School of Social Service Administration is therefore the successor of the Chicago School of Civics and the Philanthropic Service Division of the School of Commerce.

The School differs from other schools in the social service field in that it offers the student not only a series of graduate professional courses but also the opportunity of combining his professional work with a wide choice of graduate courses in the Social Service departments of a great University.

Source: University of Chicago. The Cap and Gown 1924, p. 220.

_________________________________

Edith Abbott’s Writings

Over 125 items in the following bibliography are accessible via the links that have been collected by the curator of Economics in the Rear-View Mirror. Alas, seven items have not (yet) been found, of which four are significant books published by University of Chicago Press and still under copyright protection.

“Wage Statistics in the Twelfth Census.” Journal of Political Economy, 12 (June 1904), 339-61. https://archive.org/details/jstor-1833345/page/n1/mode/2up

Review of Elements of Political Economy by J. Shield Nicolson. In School Review 12 (Nov. 1904), 754-755. https://archive.org/details/jstor-1075897/page/n1/mode/2up

Review of Trade Unionism and British Industry by Edwin A. Pratt. Journal of Political Economy 13 (Dec. 1904), 129-132. https://archive.org/details/paper-doi-10_1086_251116

Review of Women in the Printing Trades: A Sociological Study, edited by J. Ramsay MacDonald. In Journal of Political Economy 13 (March 1905), 299-303. https://archive.org/details/paper-doi-10_1086_251145/mode/2up

“Are Women Business Failures?” Harper’s Weekly, 49 (Apr. 8, 1905) Issue 2520, 496. https://archive.org/details/sim_harpers-weekly_1905-04-08_49_2520/page/496/mode/2up

“Wages of Unskilled Labor in the United States, 1850-1900.” Journal of Political Economy, 13 (June 1905), 321-67. (Ph.D. Dissertation) https://archive.org/details/jstor-1819499/page/n1/mode/2up

Review of Labor Organization among Women by Belva Mary Herron. In Journal of Political Economy 13 (Sept. 1905), 605-607. https://archive.org/details/jstor-1817853/page/n1/mode/2up

“Harriett Martineau and the Employment of Women in 1836.” Journal of Political Economy, 14 (Dec. 1906), 614-26. https://archive.org/details/jstor-1819994/page/n1/mode/2up

“Employment of Women in Industries: Twelfth Census Statistics.” Journal of Political Economy, 14 (Jan. 1906), 14-40 (with Breckinridge). https://www.jstor.org/stable/1817279

Review of Trade Unions by Geoffrey Drage. In Journal of Political Economy 14 (Jan. 1906), 53-56. https://archive.org/details/jstor-1817284/page/n1/mode/2up

Review of The Clothing Industry in New York by Jesse Eliphalet Pope. In Journal of Political Economy 14(April 1906), 252-254. https://archive.org/details/jstor-1817940/page/n1/mode/2up

“The History of Industrial Employment of Women in the United States: An Introductory Study.” In  Journal of Political Economy, 14 (Oct. 1906), 461-501. https://archive.org/details/jstor-1817741/page/n1/mode/2up

“Woman Suffrage Militant: The New Movement in England.” The Independent, 61 (Nov. 29, 1906), 1276-78. https://archive.org/details/sim_independent_1906-11-29_61_3026/page/1276/mode/2up

“Employment of Women in Industries: Cigar Making — Its History and Present Tendencies.” Journal of Political Economy, 15 (Jan. 1907), 1-25. https://archive.org/details/jstor-1817494/page/n1/mode/2up

“Municipal Employment of Unemployed Women in London.” Journal of Political Economy, 15 (Nov. 1907), 513-30. https://archive.org/details/jstor-1819109/page/n1/mode/2up

Review of Women’s Work and Wages: A Phase of Life in an Industrial City by Edward Cadbury et al. In Journal of Political Economy 15 (Nov. 1907), 563-565.  https://archive.org/details/jstor-1819119/page/n1/mode/2up

“Women in Manufactures: A Supplementary Note,” Journal of Political Economy, 15 (Dec. 1907), 619-24 (with Breckinridge and Anne S. Davis). https://archive.org/details/jstor-1820425/page/n1/mode/2up

“A Study of the Early History of Child Labor in America.” American Journal of Sociology, 14 (Jul. 1908), 15-37. https://archive.org/details/jstor-2762758/page/n1/mode/2up

“English Working Women and the Franchise.” Atlantic, 102 (Sept 1908), 343-46. https://archive.org/details/sim_atlantic_1908-09_102_3/page/342/mode/2up

“The Public Moralist and the Working Woman. Association of Collegiate Alumnae Magazine, III.18 (Dec. 1908), 12-18. https://hdl.handle.net/2027/iau.31858028383911?urlappend=%3Bseq=188%3Bownerid=13510798903605987-200

“History of the Employment of Women in the American Cotton Mills.” Journal of Political Economy:

Part I. 16 (Nov. 1908), 602-21; https://archive.org/details/jstor-1820913/page/n1/mode/2up

Part II. 16 (Dec. 1908), 680-92; https://archive.org/details/jstor-1821966/page/n1/mode/2up

Part III. 17 (Jan. 1909), 19-35. https://archive.org/details/jstor-1819902/page/n1/mode/2up

“Women in Industry: The Manufacture of Boots and Shoes.” American Journal of Sociology, 15 (Nov. 1909), 335-60. https://archive.org/details/jstor-2762515/page/n1/mode/2up

Review of Where Shall She Live? The Homelessness of the Woman Worker by Mary Higgs and Edward E. Hayward. In American Journal of Sociology 16 (Sept. 1910), 272-273. https://archive.org/details/jstor-2763060/page/n1/mode/2up

Review of Frederick William Maitland by H.A.L. Fisher. In Journal of Political Economy 18 (Nov. 1910), 750-751. https://archive.org/details/jstor-1820690/page/n1/mode/2up

Women in Industry. A Study of American Economic History. New York: Appleton and Co., 1910. https://archive.org/details/WomenInIndustryStudy/page/n4/mode/1up   

The Housing Problem in Chicago. (with Breckinridge). (parts I, VI-X written by others)

  1. Introductory Note to “Housing of Non-Family Groups of Working Men” by Milton B. Hunt. American Journal of Sociology, 16 (Sept. 1910),145-146 (with Breckinridge). https://archive.org/details/jstor-2763051/page/n1/mode/2up
  2. “Families in Furnished Rooms.” American Journal of Sociology, 16 (Nov. 1910), 289-308 (with Breckinridge). https://archive.org/details/jstor-2763087/page/n1/mode/2up
  3. “Back of the Yards.” American Journal of Sociology, 16 (Jan. 1911), 433-68 (with Breckinridge). https://archive.org/details/jstor-2763005/page/n1/mode/2up
  4. “The West Side Revisited.” American Journal of Sociology, 17 (July 1911), 1-34 (with Breckinridge). https://archive.org/details/jstor-2763253/page/n1/mode/2up
  5. “South Chicago at the Gates of the Steel Mills.” American Journal of Sociology, 17 (Sept. 1911), 145-76 (with Breckinridge). https://archive.org/details/jstor-2762945/page/n1/mode/2up

“English Poor-Law Reform.” Journal of Political Economy, 19 (Jan. 1911), 47-59. https://archive.org/details/jstor-1820483/page/n1/mode/2up

Review of Child Labor Legislation in Europe by C.W.A. Vedite. In American Economic Review (March 1911), 110-112. https://archive.org/details/jstor-1802931/page/n1/mode/2up

Finding Employment for Children Who Leave the Grade Schools to Go to Work: Report to the Chicago Woman’s Club, the Chicago Association of Collegiate Alumni, and the Women’s City Club. Chicago: Hollister Press, 1911 (with Breckinridge and Anne S. Davis). https://hdl.handle.net/2027/uc2.ark:/13960/t33205206

“Women in Industry: The Chicago Stockyards.” Journal of Political Economy, 19 (Oct. 1911), 632-54 (with Breckinridge). https://archive.org/details/jstor-1819424/page/n1/mode/2up

Review of The Solution of the Child Labor Problem by Scott Nearing, in American Economic Review (Dec. 1911), 846. https://archive.org/details/jstor-1806913/page/n1/mode/2up

The Delinquent Child and the Home. New York: Russell Sage Foundation, 1912 (with Breckinridge). https://archive.org/details/cu31924030383214/page/n7/mode/2up

Review of Wages in the United States, 1908-1910 by Scott Nearing. In Journal of Political Economy 20 (May 1912), 529-531. https://archive.org/details/jstor-1822107/page/n1/mode/2up

Review of The Living Wage of Women Workers: A Study of Incomes and Expenditures of Four Hundred and Fifty Women Workers in the City of Boston by Louise Marion Bosworth. In American Economic Review (June 1912), 380-382. https://archive.org/details/jstor-1827614/page/n1/mode/2up

Review of The Prevention of Destitution by Sidney and Beatrice Webb. In Journal of Political Economy 20 (July 1912), 754-756. https://archive.org/details/jstor-1820154/page/n1/mode/2up

Review of Making Both Ends Meet: The Income and Outlay of New York Working Girls by Sue Ainslie Clark and Edith Wyatt. In American Economic Review (September 1912), 652-654. https://archive.org/details/jstor-1804628/page/n1/mode/2up

“The First Chief of the Children’s Bureau.” Life and Labor, 2 (Oct. 1912), 299-301. https://hdl.handle.net/2027/coo.31924069101354?urlappend=%3Bseq=343%3Bownerid=27021597770090515-357

Wage-earning Woman and the State: A Reply to Miss Minnie Bronson. Boston: Equal Suffrage Association for Good Government, 1912 (with Breckinridge). https://archive.org/details/wageearningwoman00abbo

“Women’s Wages in Chicago: Some Notes on Available Data.” Journal of Political Economy, 21 (Feb. 1913), 143-58. https://archive.org/details/jstor-1819961/page/n1/mode/2up

Review of Progress and Uniformity in Child-Labor Legislation. A Study in Statistical Measurement by William F. Ogburn. In American Economic Review, 3 (June 1913), 397-399. https://archive.org/details/jstor-1827991/page/n1/mode/2up

“Public Pensions to Widows and Children.” American Economic Review, 3 (June 1913), 473-78. https://archive.org/details/jstor-1828023/page/n27/mode/2up

Reviews of The American Girl in the Stockyards District by Louise Montgomery; Women in Trade Unions in San Francisco by Lillian R. Matthews; Artifical Flower Makers by Mary Van Kleeck. In American Economic Review (March 1914), 164-166. https://archive.org/details/jstor-1805013/page/n1/mode/2up

“A Forgotten Minimum Wage Bill.” Life and Labor, 5 (Jan. 1915), 13-16. https://hdl.handle.net/2027/mdp.39015010962978?urlappend=%3Bseq=21%3Bownerid=13510798887191435-25

“Progress of the Minimum Wage in England.” Journal of Political Economy, 23 (Mar. 1915), 268-77. https://archive.org/details/jstor-1819662/page/n1/mode/2up

Review of Minimum Rates in the Chain-making Industry (Studies in the Minimum Wage, No. 1) by R. H. Tawney. Journal of Political Economy 23 (April 1915), 400-401. https://archive.org/details/jstor-1819291/page/n1/mode/2up

“Statistics Relating to Crime in Chicago.” In Report of the City Council Committee of Chicago on Crime in the City of Chicago, pp. 17-88. Chicago: City Council Committee, 1915. https://archive.org/details/reportofcitycoun00chic/page/16/mode/2up

“The Copycat Vote.” New Republic, 2 (Apr. 24, 1915), 304. https://hdl.handle.net/2027/hvd.hxqfnz?urlappend=%3Bseq=366%3Bownerid=27021597764513068-380

“Education for Social Work.” In Department of Interior, Bureau of Education, Report of the Commissioner of Education for the Year Ended June 30, 1915, vol. 1 (Washington: Government Printing Office, 1915). https://archive.org/details/reportofcommissi00unit_51/page/344/mode/2up

“Field-Work and the Training of the Social Workers.” In Proceedings of The National Conference of Charities and Correction at the Forty-Second Annual Session held in Baltimore, Maryland, May 12-19, 1915, pp. 615-21. Chicago: Hildmann Printing Co., 1915. https://archive.org/details/sim_national-conference-on-social-welfare-bulletin_may-12-19-1915_42/page/614/mode/2up

“Statistics in Chicago Suffrage.” New Republic, 3 (June 12, 1915), 151. https://hdl.handle.net/2027/mdp.39015086341164?urlappend=%3Bseq=191%3Bownerid=13510798902096126-219

“Are Women a Force for Good Government?” National Municipal Review, 4 (July 1915), 437-447. https://hdl.handle.net/2027/hvd.32044106250632?urlappend=%3Bseq=497%3Bownerid=27021597765335525-549

The Real Jail Problem. Chicago: Juvenile Protective Association of Chicago, 1915. https://archive.org/details/realjailproblem00abbo/page/n3/mode/2up

The One Hundred and One County Jails of Illinois and Why They Ought to Be Abolished. Chicago: Juvenile Protective Association of Chicago, 1916. https://archive.org/details/onehundredonecou00abbo/page/n3/mode/2up

“Cheap Clothes and Nasty.” New Republic, 4 (Jan. 1, 1916), 217-219. https://hdl.handle.net/2027/hvd.hxqfp4?urlappend=%3Bseq=274%3Bownerid=27021597764513482-278

“The Woman Voter and the Spoils System in Chicago.” National Municipal Review, 5 (July 1916), 460-465. https://hdl.handle.net/2027/hvd.32044106250624?urlappend=%3Bseq=498%3Bownerid=27021597765323744-544

Review of Women in Modern Industry by B. L. Hutchins. In American Economic Review (June 1916), 399-400. https://archive.org/details/jstor-1813274/page/n1/mode/2up

Review of Summary of the Report on Condition of Woman and Child Wage Earners in the United States, Bulletin of the U.S. Bureau of Labor Statistics. In American Economic Review, 662-664. https://archive.org/details/jstor-1808551/page/n1/mode/2up

Review of Old Age Pensions: Their Actual Working and Ascertained Results in the United Kingdom by H. J. Hoare. American Journal of Sociology (Sept. 1916), 277-278. https://archive.org/details/jstor-2763833/page/n1/mode/2up

“Administration of the Illinois Funds-to-Parents Laws.” United States Department of Labor Bulletin 212, pp. 818-34. Washington: Government Printing Office, 1917. https://hdl.handle.net/2027/uc1.a0004011193?urlappend=%3Bseq=980%3Bownerid=13510798903282064-1000

“The Experimental Period of Widows Pension Legislation.” In Proceedings of The National Conference of Social Work, 1917, pp. 154-65. https://archive.org/details/sim_national-conference-on-social-welfare-bulletin_june-6-13-1917_44/page/154/mode/2up

“Charles Booth, 1840-1916.” Journal of Political Economy, 25 (Feb 1917), 195-200. https://archive.org/details/jstor-1819721/page/n1/mode/2up

“The War and Women’s Work in England.” Journal of Political Economy, 15 (July 1917), 641-678. https://archive.org/details/jstor-1821772/page/n1/mode/2up

“Field Work Training with Social Agencies.” In Report of the Association of Urban Universities, November, 1917, pp. 92-103. Concord, N.H.: Rumford Press, 1917-18. https://hdl.handle.net/2027/iau.31858045945593?urlappend=%3Bseq=274%3Bownerid=117203284-282

Truancy and Non-Attendance in the Chicago Schools: A Study of the Social Aspects of the Compulsory Education and Child Labor Legislation of Illinois. Chicago: University of Chicago Press, 1917 (with Breckinridge). https://archive.org/details/truancynonattend00abbo/page/n5/mode/2up

Democracy and Social Progress in England. University of Chicago War Papers, 8. Chicago: University of Chicago Press, 1918. https://archive.org/details/democracysocialp00abbo/mode/2up

“The Social Case Worker and the Enforcement of Industrial Legislation.” In Proceedings of The National Conference of Social Work, 1918, pp. 312-19. https://archive.org/details/sim_national-conference-on-social-welfare-bulletin_may-15-22-1918_45/page/312/mode/2up

“Pensions, Insurance and the State.” In Proceedings of The National Conference of Social Work, 1918, pp. 388-89. https://archive.org/details/sim_national-conference-on-social-welfare-bulletin_may-15-22-1918_45/page/388/mode/2up

“Crime and the War.” Journal of the American Institute of Criminal Law and Criminology, 9 (May 1918), 32-45. https://archive.org/details/jstor-1133731/page/n1/mode/2up

Review of three works on women workers. In American Journal of Sociology 23 (Jan. 1918), 551-552. https://archive.org/details/jstor-2763523/page/n1/mode/2up

Reviews of six books on women and war work. In American Economic Review (Dec. 1918), 819-824.https://archive.org/details/jstor-1803702/page/n1/mode/2up

“Health Insurance in Great Britain.” In Report of the Health Insurance Commission of the State of Illinois, May 1, 1919, pp. 600-624. Springfield: Illinois State Journal Co., 1919. Also in Report of the Ohio Health and Old Age Insurance Commission, February, 1919, pp. 312-40. Columbus: F. J. Heer Printing Co., 1919. https://archive.org/details/cu31924002406951/page/600/mode/2up

“Probation and Suspended Sentence” (Report of Committee “B” of the Institute). Journal of the American Institute of Criminal Law and Criminology, 10 (Nov. 1919), 341-50. https://archive.org/details/jstor-1133813/page/n1/mode/2up

Review of Report of the War Cabinet Committee on Women in Industry in American Economic Review (June 1920), 358-362. https://archive.org/details/jstor-1804881/page/n1/mode/2up

Review of Italian Emigration of our Times by Robert F. Foerster. In American Political Science Review 14 (Aug. 1920), 523-524. https://archive.org/details/jstor-1946285/page/n1/mode/2up

The Administration of the Aid-to-Mothers Law in Illinois. U.S. Children’s Bureau. Washington: U.S. Government Printing Office, 1921 (with Breckinridge).https://archive.org/details/administrationof00abbo/page/n3/mode/2up

Review of The Passing of the County Jail: Individualization of Misdemeanants through a Unified Correctional System by Stuart Alfred Queen. The American Journal of Sociology (May 1921), 792-793. https://archive.org/details/jstor-2764348

“The Promise and Practice of Social Legislation.” University Journal (alumni edition, University of Nebraska), 17 July 1921), 4-11. https://hdl.handle.net/2027/mdp.39015080407714?urlappend=%3Bseq=6%3Bownerid=13510798897152302-10

“Police Brutality in Chicago.” The Nation, 114 (Mar. 8, 1922), 286-87. https://hdl.handle.net/2027/pst.000068744618?urlappend=%3Bseq=308%3Bownerid=13510798902987282-322

“Tragedy of the Excess Quota.” New Republic, 30 (Mar. 8, 1922), 52-53. https://hdl.handle.net/2027/hvd.hwqwpg?urlappend=%3Bseq=66%3Bownerid=27021597767357933-72

Review of Immigration and the Future and The Federal Administration and the Alien both by Frances Kellor. In Journal of Political Economy 30 (Apr. 1922), 312-314. https://archive.org/details/jstor-1822697/page/n1/mode/2up

Discussion of “Immigration under the Percentum Limit Law,” by W. W. Husband. In Proceedings of The National Conference of Social Work, 1922, pp. 463-66. https://archive.org/details/sim_national-conference-on-social-welfare-bulletin_june-22-29-1922_49/page/462/mode/2up

What the Women of Illinois Ought to Know and Ought to Do about the Questions of Social Hygiene: A Report Submitted to the Committee Appointed at the Request of the Joint Conference of the Women’s Clubs of Chicago, 1922.

“Recent Statistics Relating to Crime in Chicago.” Journal of the American Institute of Criminal Law and Criminology, 13 (Nov. 1922), 329-58. https://archive.org/details/jstor-1133931/page/n1/mode/2up

“Training in Case Work and Special Administrative Problems in a University.” In The Social Service of the Courts: Proceedings of the Sixteenth Annual Conference of The National Probation Association, 1922, pp 59-68. New York: National Probation Association, 1923. https://hdl.handle.net/2027/njp.32101067578383?urlappend=%3Bseq=65%3Bownerid=27021597769832968-69

Review of The History of Public Poor Relief in Massachusetts, 1620-1920. In American Journal of Sociology(Nov. 1922), 364-366. https://archive.org/details/jstor-2764686/page/n1/mode/2up

“The English Census of 1921.” Journal of Political Economy, 30 (Dec. 1922), 827-40. https://archive.org/details/jstor-1822472/page/n1/mode/2up

“Is One Per Cent in Quarantine a Public Health Measure?” Illinois League of Women Voters Bulletin, 3 (1923), 7-9.

Review of Penology in the United States by Louis N. Robinson. In American Journal of Sociology 29 (July 1923), 105-106. https://www.jstor.org/stable/2764420

“Federal Immigration Policies, 1864-1924.” University Journal of Business, 2 (1924), (Mar. 1924), 133-56; (Jun. 1924), 347-67; (Sep. 1924), 455-80. https://www.jstor.org/stable/2354831 ; https://www.jstor.org/stable/2354665 ; https://www.jstor.org/stable/2354651

“Immigration Legislation and the Problems of Assimilation.” In Proceedings of The National Conference of Social Work, 1924, pp. 82-91. https://archive.org/details/sim_national-conference-on-social-welfare-bulletin_june-25-july-2-1924_51/page/82/mode/2up

Immigration: Select Documents and Case Records. Chicago: University of Chicago Press, 1924. https://hdl.handle.net/2027/mdp.39015010521006

“English Statistics of Pauperism during the War.” Journal of Political Economy, 33 (Feb. 1925), 1-32. https://www.jstor.org/stable/1821974

Historical Aspects of the Immigration Problem. Chicago: University of Chicago Press, 1926. https://hdl.handle.net/2027/mdp.39015008786579

“Training for the Policewoman’s Job.” Woman Citizen, 10 (Apr 1926), 30. https://archive.org/details/sim_womans-journal_1926-04_10_13/page/30/mode/2up

“The Civil War and the Crime Wave of 1865-70.” Social Service Review, 1 (June 1927), 212-34. https://archive.org/details/sim_social-service-review_1927-06_1_2/page/212/mode/2up

“The Webbs on the English Poor Law.” Social Service Review, 3 (June 1929), 252-69. https://archive.org/details/sim_social-service-review_1929-06_3_2/page/252/mode/2up

Report on Crime and Criminal Justice in Relation to the Foreign Born, National Commission on Law Observance and Enforcement (Wickersham Commission). No. 10. Washington: Government Printing Office, 1931. https://hdl.handle.net/2027/uc1.b4628567

Social Welfare and Professional Education. Chicago: University of Chicago Press, 1931, and ed. 1942.

“Poor People in Chicago.” New Republic, 72 (Oct. 5, 1932), 209. https://archive.org/details/sim_new-republic_1932-10-05_72_931/page/208/mode/2up

“The Fallacy of Local Relief.” New Republic, 72 (Nov. 9, 1932), 348-50. https://archive.org/details/sim_new-republic_1932-11-09_72_936/page/347/mode/2up

“The Crisis in Relief.” The Nation, 137 (Oct. 11, 1933), 400-402. https://archive.org/details/sim_nation_1933-10-11_137_3562/page/400/mode/2up

“Abolish the Pauper Laws.” Social Service Review, 8 (Mar. 1934), 1-16. https://archive.org/details/sim_social-service-review_1934-03_8_1

“Don’t Do It, Mr. Hopkins!” The Nation, 140 (Jan. 9, 1935), 41-42. https://archive.org/details/sim_nation_1935-01-09_140_3627/page/40/mode/2up

“Evictions during the Chicago Rent Moratorium Established by the Relief Agencies, 1931-1933.” Social Service Review, 9 (Mar. 1935), 34-57 (with Katherine Kiesling). https://archive.org/details/sim_social-service-review_1935-03_9_1/page/34/mode/2up

“The Pauper Laws Still Go On.” Social Service Review, 9 (Dec. 1935), 731-56. https://archive.org/details/sim_social-service-review_1935-12_9_4/page/730/mode/2up

“Jane Addams Memorial Service.” In Proceedings of The National Conference of Social Work, 1935, pp. 3-5. https://archive.org/details/sim_national-conference-on-social-welfare-bulletin_june-09-15-1935/page/2/mode/2up

The Tenements of Chicago, 1908-1935. Chicago: University of Chicago Press, 1936 (with Breckinridge).

“Federal Relief Sold Down the River.” The Nation, 142 (Mar. 18, 1936), 346. https://archive.org/details/sim_nation_1936-03-18_142_3689/page/346/mode/2up

“Training for the Public Welfare Services.” Public Welfare News, 4 (Mar. 1936), 5.

“Public Welfare and Politics.” In Proceedings of The National Conference of Social Work, 1936, pp. 27-45; https://archive.org/details/sim_national-conference-on-social-welfare-bulletin_may-18-23-1936/page/26/mode/2up  also in Social Service Review, 10 (Sept. 1936), 395-412. https://archive.org/details/sim_social-service-review_1936-09_10_3

“Public Assistance—Whither Bound?” In Proceedings of The National Conference of Social Work, 1937, pp. 3-25. https://archive.org/details/sim_national-conference-on-social-welfare-bulletin_may-23-29-1937/page/n11/mode/2up

Some American Pioneers in Social Welfare: Select Documents with Editorial Notes. Chicago: University of Chicago Press, 1937.

“Is There a Legal Right to Relief?” Social Service Review, 12 (June 1938), 260-75. https://archive.org/details/sim_social-service-review_1938-06_12_2/page/260/mode/2up

“Poor Law Provision for Family Responsibility.” Social Service Review, 12 (Dec. 1938), 598-618. https://archive.org/details/sim_social-service-review_1938-12_12_4/page/598/mode/2up

“A Sister’s Memories.” Social Service Review, 13 (Sept. 1939), 351-408. https://archive.org/details/sim_social-service-review_1939-09_13_3

“Unemployment Relief a Federal Responsibility.” Social Service Review, 14 (Sept. 1940), 438-52. https://archive.org/details/sim_social-service-review_1940-09_14_3/page/438/mode/2up

“Relief, the No Man’s Land, and How to Reclaim It.” In Proceedings of The National Conference of Social Work, 1940, pp. 187-98. https://archive.org/details/sim_national-conference-on-social-welfare-bulletin_may-26-june-01-1940/page/186/mode/2up

Public Assistance. Chicago: University of Chicago Press, 1940. Vol. I [Note: very incomplete copy at archive.org] ; Vol. II https://archive.org/details/dli.ernet.544099/page/n7/mode/2up

United States, 76th Cong., 3rd Sess., House, Select Committee to Investigate the Interstate Migration of Destitute Citizens, Aug. 19, 20. and 21, 1940, pp. 1179-90. https://archive.org/details/interstatemigrat03unit/page/1178/mode/2up

“Work or Maintenance: A Federal Program for the Unemployed.” In Proceedings of The National Conference of Social Work, 1941, pp. 332-43 https://archive.org/details/sim_national-conference-on-social-welfare-bulletin_june-01-07-1941/page/332/mode/2up ; revised in Social Service Review, 15 (Sept. 1941), 520-32. https://archive.org/details/sim_social-service-review_1941-09_15_3/page/520/mode/2up

“Twenty-one Years of University Education for the Social Services, 1920-41.” A Report to the Alumni with a Register of Alumni Who Received Higher Degrees, 1920-1942, and Their Dissertation Subjects. Social Service Review, 15 (Dec. 1941), 670-705. https://archive.org/details/sim_social-service-review_1941-12_15_4/page/670/mode/2up

“Juvenile Delinquency during the First World War, Notes on the British Experience 1914-1918.” Social Service Review, 17 (June 1943), 192-212. https://archive.org/details/sim_social-service-review_1943-06_17_2/page/192/mode/2up

“Some Charitable Bequests in Early English Wills (1284-1580) and Statutes (1414-1601) to Protect Charitable Gifts.” Social Service Review, 20 (June 1946), 231-46. https://archive.org/details/sim_social-service-review_1946-06_20_2/page/230/mode/2up

“Three American Pioneers in International Social Welfare.” The Compass, 28 (May 1947), 6.

“Work of Thomas H. Gallaudet and the Teaching of the Deaf.” Social Service Review, 21 (Sept. 1947), 375-86. https://archive.org/details/sim_social-service-review_1947-09_21_3/page/374/mode/2up

“Sophonisba P. Breckinridge Over the Years.” Social Service Review, 22 (Dec. 1948), 417-23. https://archive.org/details/sim_social-service-review_1948-12_22_4/page/n7/mode/2up

“Grace Abbott and Hull-House, 1908-21. Social Service Review 24, Part I, (Sept. 1950), 374-94. https://archive.org/details/sim_social-service-review_1950-09_24_3/page/374/mode/2up; and Part II, (Dec. 1950), 493-518. https://archive.org/details/sim_social-service-review_1950-12_24_4/page/492/mode/2up

“The Survey Award: Acceptance Speech.” In Proceedings of The National Conference of Social Work, 1951, pp. ix-x. https://quod.lib.umich.edu/n/ncosw/ACH8650.1951.001/16?rgn=full+text;view=image

“The Hull-House of Jane Addams.” Social Service Review, 26 (Sept. 1952), 334-38. https://archive.org/details/sim_social-service-review_1952-09_26_3/page/334/mode/2up

Sources:
Rachel Marks, The Published Writings of Edith Abbott: A Bibliography, American Journal of Sociology 32 (March 1958), 51-56;
Lela B. Costin (1983). Two Sisters for Social Justice: A Biography of Grace and Edith Abbott. Urbana and Chicago: University of Illinois Press, pp. 287-293.

Image Source: Portrait of Edith Abbott by Melvin H. Sykes (1919). University of Chicago Photographic Archive, apf1-00004, Hanna Holborn Gray Special Collections Research Center, University of Chicago Library. Image colorized by Economics in the Rear-view Mirror.

Categories
Exam Questions Harvard Industrial Organization

Harvard. Railways and public utility regulation. Exam questions. Meyer, 1902-1903.

Besides adding to the database of economics examination questions, this post provides us with additional biographical information for the railroad and municipal utility expert, Harvard economics Ph.D. alumnus Hugo Richard Meyer (1866-1923). He was an assistant professor of economics at the University of Chicago in 1903-05.

______________________

From a review of Meyer’s book,
Government Regulations of Railway Rates

Hugo Richard Meyer is a native of Cincinnati, O., and is now 39 years of age. He attended the pubic schools of that city in early childhood and in 1877 he was taken by his parents to Germany, where he remained five years. From Germany, his father returned to America and made his home in Denver, Colo., where in the high school young Meyer prepared for Harvard college, from which he was graduated in 1892. Afterward he attended Harvard Graduate school five years.

Prof. Meyer’s usefulness began as an instructor of political economy in Harvard, serving from 1897 until 1903. In 1903 he was one of three commissioners appointed by the Governor to investigate and report on the wisdom of amending the laws of Massachusetts, governing the exercise of the right of eminent domain by cities in public improvement. In 1903, he wrote a series of leaflets on the subject of municipal ownership, taking the side of private ownership. Mr. Meyer began the study of economics with a prejudice in favor of socialism, but came out in a strong belief of individualism. He has studied socialism in Australia and the operation of municipal ownership in Great Britain. He especially has paid attention to the government regulation of railroad rates in the United States, Germany, France, Austria Hungary, Russia and Australia. In his book, Government Regulation of Railway Rates [a study of the experience of the United States, Germany, France, Austria-Hungary, Russia, and Australia] , he gives a comparative study of the experiece of public regulation of railway rates in countries where he has been.

In addition to this book, Mr. Meyer has two other works in preparation. One deals with municipal ownership in Great Britain [followed by The British State Telegraphs (1907) and Public Ownership and the Telephone in Great Britain (1907)] and the other with state socialism in Australia. Mr. Meyer has written also articles on the regulation of railway rates in Europe, and an article entitled, “Rate Making by Government,” which were published respectively in 1903 and 1905.

Mr. Meyer came into national prominence in May last, when he appeared before the Senate sub-committee of interstate commerce and gave his views on the question of rate making by government. It was the clearest statement given by any witness on either side of this great question. He presented an array of facts that no other witness had summoned, and when he left the chair it was the judgment of all that he had been the most impressive witness that had appeared before the committee.

SourceThe Topeka Daily Herald, October 14, 1905, p. 8.

______________________

Course Description
Economic 5
1902-1903

  1. 1hf. Railways and other Public Works. under Public and Corporate Management. Half-course (first half-year). Tu., Th., Sat., at 1.30. Mr. Meyer.

This course reviews the history and working of different modes of dealing with transportation, and deals with the questions of street railways, gas, and electric light supply.

The manner in which Germany [Municipal Ownership in Germany. Journal of Political Economy, (1906), 14(9), 553–567.], France [The Breakdown of State Railway Building in France. Journal of Political Economy (1906), 14(7), 450–453.], and Russia have regulated railway rates, either by exercising control over private corporations, or by assuming public ownership and operation, will be studied with special reference to the effect of such regulation upon the elasticity of railway rates and the ability of the railways to develop trade and industry. In this connection will be studied the part played by the railways and by the waterways in the development of the leading industries of Germany, France, and Russia; as well as the question why Germany, France, and Russia are obliged to have recourse to the waterways for the performance of services that in the United States are rendered by the railways.

The attempts of the railways of the United States to regulate railway rates through pools will be compared with the attempts of the several states and of the federal government to regulate rates through legislation and through commissions. Typical decisions of pools, of state commissions, and of the Interstate Commerce Commission will be studied for the purpose of ascertaining whether those decisions are founded on a body of principles that may be said to have the character of a science, or whether they express merely the judgment of administrative officers on questions of fact to which no body of scientific principles can be made to apply. In conclusion, the question whether railway rates should be regulated through pools or through legislation and commissions will be discussed by means of a comparison of the experience of the European countries, the United States under the régime of regulation by pools, and the United States under the régime of regulation by state and federal legislation and by commissions.

The problem of the public ownership and operation of the railways will be discussed under the following heads: the difficulties experienced by Prussia, France, Italy, Russia, and the Australian Colonies, in making the railway budget fit into the state budget; the problem of a large body of civil servants in a self-governing community, as illustrated in the experience of the Australian Colonies.

The question of the regulation and control of private corporations operating street railways, gas, and electric light plants will be studied by means of a review of the experience of Massachusetts, which exercises control by means of legislation and commissions; and the experience of Great Britain, which exercises control by means of legislation, and, in many instances, supplements that control by the policy of municipal ownership.

______________________

Enrollment
Economics 5
1902-1903

Economics 5. 1hf. Mr. Meyer. — Railways and other Public Works under Public and Corporate Management.

Total 60: 1 Gr., 30 Se., 14 Ju., 5 So., 1 Fr., 9 Others.

Source: Harvard University. Annual Report of the President of Harvard College, 1902-03, p. 68.

______________________

ECONOMICS 5
Mid-year Examination
1902-1903

Give one hour to question 1

  1. If the Act to Regulate Commerce has been enacted at the close of the Civil War, and if it had been interpreted and enforced in the spirit in which the Interstate Commerce Commission has interpreted it, what, in your opinion, would have been the effect upon the development of the resources of the United States; and what would be the present position of affairs as to local discriminations and personal discriminations? Indicate briefly the facts upon which you base your opinion, but do not recite the details of those facts.
  2. If you were asked to ascertain with what efficiency and reasonableness of charges the railways of a country were serving that country, what information should you seek to obtain?
  3. What facts, and what statistics should you use, were you asked to discuss the question whether the “basing-point” system concentrates trade and population?
  4. Among the dicta of the Interstate Commerce Commission is this one, that the railways have no right to drive out of business the vessels upon the rivers. Discuss the soundness of this dictum, basing your discussion upon the experience of several countries.
  5. The Northern Pacific Railroad Co. charges 65 cents per 100 Ibs. for carrying Pacific slope sugar from the Pacific slope to St. Paul. That charge is fixed by the lake and rail route which carries Atlantic sea-board sugar from the Atlantic sea-board to St. Paul. The N. P. R. Co. charges 97 cents per 100 lbs. for carrying sugar from the Pacific slope to Fargo, which is 240 miles west of St. Paul, on the direct line between the Pacific slope and St. Paul. It also charges 32 cents per 100 lbs. for carrying sugar from St. Paul to Fargo.
    The distance from New York to St. Paul is 1373 miles; from San Francisco to St. Paul, 2300 miles. The charge of 97 cents from San Francisco to Fargo is reasonable per se.
    The San Francisco output of refined sugar is 140,000 tons to 150,000 a year; and of that output, between 30,000 tons and 40,000 are sold to St. Paul, the balance being sold to points upon the Pacific slope and points in the mountain districts between the Pacific slope and St. Paul.
    M. Raworth, a wholesale grocer at Fargo, complains that the rates charged by the N. P. R. Co. are in violation of Clause 4 of the Act to Regulate Commerce.
    What, in your opinion, would be (1) the decision of the Interstate Commerce Commission; (2) the decision of the Supreme Court of the United States?
    Base your opinion upon decisions made respectively by the Commission and the Supreme Court.
  6. State the conditions under which the Interstate Commerce Commission said that certain cuts in the rates on “grain for export” were “a gratuitous gift to the foreigner”; and criticize the reasoning with which the Commission sought to support its position.
  7. When it was suggested in England that the Justices of the Court of Common Pleas be constituted a railway tribunal, Lord Campbell, afterward Lord Chancellor, in behalf of his associates declined the responsibility, saying: “This is not a code which the judges can interpret, it leaves them altogether to exercise their discretion as to what they may deem reasonable. They are, besides, to form a just judgment on all matters of complaint relating to railway management that may come before them, and they are to lay down a code of regulation for the government of railway companies. The judges, himself among the number, feel themselves incompetent to decide on these matters. He had spent a good part of his life in studying the laws of his country, but, he confessed, he was wholly unacquainted with railway management, as well as the transit of goods by boat. He knew not how to determine what is a reasonable rate…”
    What judicial opinion does this quotation recall to your mind; and what is the significance of that opinion?
  8. A recent Siberian Census revealed the fac that there were large and fertile regions in Siberia in which the farmers cultivated, on an average, sixteen acres of land. The farmers in question had emigrated from European Russia in the seventies. What do you presume to be the reasons for that state of affairs?

Source: Harvard University Archives. Mid-year Examinations 1852-1943. Box 6. Papers (in the bound volume Examination Papers Mid-years 1902-1903).
Also included in: Harvard University Archives. Examination Papers 1873-1915. Box 6. Papers Set for Final Examinations in History, Government, Economics, History of Religions, Philosophy, Education, Fine Arts, Architecture, Landscape Architecture, Music in Harvard College, June 1903 (in the bound volume Examination Papers 1902-1903).

Image Source: A newspaper sketch of Hugo Richard Meyer from 1905 reproduced in Wikimedia Commons.

Categories
Chicago Curriculum Statistics

Chicago. Report of the Committee on Mathematical Statistics. Henry Schultz, 1938

 

The following report on the work of the Committee on Mathematical Statistics by economics professor Henry Schultz to President Robert M. Hutchins of the University of Chicago was written shortly before he left Chicago to go on sabbatical leave at the University of California at Los Angeles. Schultz had just published his magnum opus, The Theory and Measurement of Demand, earlier that Spring. 

Henry Schultz, his wife and both daughters tragically died November 26, 1938 in a horrific automobile accident about sixty miles east of San Diego on U.S. highway 80, near Laguna Junction. 

_____________________________

The University of Chicago
Department of Economics

Aug. 6, 1938

President Robert M. Hutchins,
University of Chicago.

Dear Mr. Hutchins:

            The Committee on Mathematical Statistics, which was organized on March 6, 1936, and which began to work in the Autumn Quarter of 1936, completed its first series of courses in the Spring Quarter of 1938. It is, therefore, appropriate that I give a brief report of our activities.

            During the last two years the Committee gave six courses in which there were enrolled a total of 104 students from seven different departments. The courses and the professors in charge were:

Course

Professor

Statistics 301—Survey of Mathematical Statistics I.
The Elements (Autumn 1936)
1 Major
Thurstone
Statistics 302—Survey of Mathematical Statistics II.
Probability and Least Squares (Winter 1937)
1 Major
Bartky
Statistics 301—Survey of Mathematical Statistics I.
The Elements (Autumn 1937)
1 Major
Bartky
Statistics 302—Survey of Mathematical Statistics II.
Probability and Least Squares (Winter 1937)
1 Major
Bartky
Statistics 311—Correlation and Curve-fitting
(Winter 1937)
2 Majors
Schultz
Statistics 312—Probability, Sampling, and Frequency Distributions
(Spring 1938)
2 Majors
Schultz and Bartky

            The courses given did not, and were not intended to, avoid such duplication as may exist in the teaching of statistics on the campus. As is clearly stated in the Committee’s announcement, the instruction for which this Committee takes a co-ordinating responsibility is intended for those who have the conventional courses in analytic geometry and in the differential and integral calculus as well as a good introductory course in statistics, preferably one given in the Department in which the student intends to do his major work.

It is this policy of the Committee which is the source of its strength and of its weakness. It is a source of strength, because the prerequisite of a course in statistics in the Department in which the student intends to do his major work, has made it clear to the various Departments that the Committee was not interfering with the courses in elementary statistics given by them and has secured for it the good will of the statisticians on the campus. The policy is also a source of weakness, because it makes the Committee dependent on the various Departments for students and for providing them with the necessary prerequisites. Unless a Department is liberal in granting credits to its students for courses taken with the Committee the student cannot, as a rule, afford to take the entire sequence of courses offered.

This is probably the most important factor in the requests which we have received for a separate degree in statistics. We believe, however that the time is not yet ripe for a serious consideration of this question. In the first place, there are no positions for “pure statisticians” except to teach other “pure statisticians,” i.e., mathematicians. The demand is generally for a statistically trained biologist, psychologist, or economist but not for a ”pure” mathematical statistician. The situation in this respect is, however, likely to change.

In the second place, we are not prepared to grant degrees even if we wished to, and had the authority of the three members constituting this Committee, one is primarily an economist, the other is primarily a psychologist, and the third is primarily a mathematical astronomer.

            The economist and the psychologist have so much to do in their respective fields that they will be compelled, before long, to give up the attempt to keep in intimate touch with the very rapid developments in probability and mathematical statistics. This would leave only one person, Professor Bartky who could be counted upon to follow the developments in mathematical statistics and probability and do research in this field. What we need, therefore, ls at least one additional mathematician who has the ability and is qualified by training and experience to make the field of statistical inference his life work, and who is also at home in at least one empirical science. We believe that Professor S. S. Wilks of Princeton University comes close to meeting excellently all these requirements. We recommend that you look into his qualifications for work on this Committee and for consultation with the various statisticians on matters falling within his field of competence.

            The field of statistical inference is expanding at a very rapid rate. The University of California, Iowa State College, the University of Iowa, Princeton University, George Washington University, and other institutions have recently appointed men to develop their work in statistics. If the University of Chicago is to continue to do distinguished work in this field it will have to attract the most promising men it can find and to provide them with favorable conditions for their creative activities.

            The Committee requests that a sum of $300 be appropriated to it for the part-time services of a qualified graduate student to assist in the preparation of lecture and text materials. This sum requested is to supplement that obtained for mimeographing from the Social Science Division. It is understood that the money will not be used unless a qualified person can be obtained for the work.

Sincerely yours,

[signed]
Henry Schultz, Chairman,
Committee on Mathematical Statistics.

HS DH

_____________________________

Carbon Copy of President Hutchins Reply

September 9, 1938

Dear Mr. Schultz:

            I have read with much interest your report of August 6 on the first series of courses given by the Committee on Mathematical Statistics. The Committee is to be congratulated on the splendid progress which has been made.

            The financial aspect of this matter will have to be deferred until preparation of the budget for the year 1939-40. Your request for an appropriation for the part-time services of a graduate student to assist in the preparation of lecture and text materials will be considered at that time.

Sincerely yours,
ROBERT M. HUTCHINS

Professor Henry Schultz
404 Social Science Research Building
FACULTY EXCHANGE

Source: University of Chicago Archives. Office of the Presdient. Hutchins Administration. Records. Box 283, Folder10, “Economics”.

Categories
Exam Questions Harvard Money and Banking

Harvard. Banking and its history. Enrollment and final exam. Sprague, 1902-1903

Money and banking were the subjects in a two semester sequence of distinct courses at Harvard in 1902-03. Material for the money course taught by Abram Piatt Andrew, Jr. can be found in the previous post. Here we have a description, enrollment figures, and the final exam questions for economics instructor O.M.W. Sprague’s banking course in the sequence.

______________________

8b2hf. Banking and the History of the leading Banking Systems. Half-course (second half-year). Mon., Wed., Fri., at 10. Dr. Sprague.

In Course 8b, after a summary view of early forms of banking in Italy, Amsterdam, and Hamburg, a more detailed account is given of the development, to the middle of the nineteenth century, of the system of banking in which notes were the principal form of credit and the chief subject of discussion and legislation. The rise and growth of the modern system of banking by discount and deposit is then described. The work is both historical and comparative in its methods. The banking development, legislation, and present practice of various countries, including England, France, Germany, Scotland, and Canada, are reviewed and contrasted. Particular attention is given to banking history and experience in this country: the two United States banks; the more important features of banking in the separate states before 1860; the beginnings, growth, operation, and proposed modification of the national banking system; and credit institutions outside that system, such as state banks and trust companies.

The course of the money markets of London, Paris, Berlin, and New York will be followed during a series of months, and the various factors, such as stock exchange dealings, and international exchange payments, which bring about fluctuations in the demand for loans, and the rate of discount upon them will be considered. In conclusion the relations of banks to commercial crises will be analyzed, the crises of 1857 and 1893 being taken for detailed study.

Written work, in the preparation of short papers on assigned topics, and a regular course of prescribed reading will be required of all students.

The course is open to those who have taken Economics 1.

Source: Harvard University. Faculty of Arts and Sciences, Division of History and Political Science [Comprising the Departments of History and Government and Economics], 1902-03. Published in The University Publications, New Series, no. 55. June 14, 1902.

______________________

Economics 8b. 2hf. Dr. Sprague. — Banking and the History of the leading Banking Systems.

Total 133: 2 Gr., 38 Se., 52 Ju., 27 So., 1 Fr., 13 Others.

Source: Harvard University. Annual Report of the President of Harvard College, 1902-03, p. 68.

______________________

ECONOMICS 8b

Answer the questions strictly in their order.

  1. (a) Why does a fall in the prices of stock exchange securities tend to reduce the loans and the deposit liabilities of the New York banks?
    (b) Why is the rate for call loans subject to greater fluctuations than those for time loans or on commercial paper?
  2. (a) Explain the relation between the supply of commercial bills of exchange and the rates for bankers’ sterling.
    (b) Explain the usual method by which a profit is realized from the export of gold from New York to Paris.
  3. Define or explain—
    1. Due to approved reserve agents.
    2. Bills payable.
    3. Cable transfers.
    4. The bank price of gold.
  4. Changes in the charter of the Reichsbank in 1899.
  5. What changes in other items would involve a change in the amount of deposits?
  6. Discuss the advantages of a central reserve bank and consider its applicability to the United States.
  7. Give some account of failures of the national banks.
  8. How do loans in Europe serve to reduce the credit liabilities of the New York banks?

Source: Harvard University Archives. Examination Papers 1873-1915. Box 6. Papers Set for Final Examinations in History, Government, Economics, History of Religions, Philosophy, Education, Fine Arts, Architecture, Landscape Architecture, Music in Harvard College, June 1903 (in the bound volume Examination Papers 1902-1903).

Image Source: Portrait of Oliver Mitchell Wentworth Sprague in the Harvard Class Album 1915, colorized by Economics in the Rear-view Mirror.

Categories
Chicago Faculty Regulations

Chicago. Economics Ph.D. requirement. Either linear algebra or foreign language reading comprehension, 1968

 

At the University of Chicago in the mid-1950s one economics Ph.D. requirement was to demonstrate an effective reading knowledge of either French or German or some other foreign language approved by the department..

The 1968 memo to graduate students transcribed below announced that the “satisfaction of either a mathematics or foreign language requirement” would be the rule for a Ph.D. in economics.

_________________________

History of Foreign Language Requirement for Ph.D.

January 12, 1968

To: Faculty, Division of the Social Sciences
From: D. Gale Johnson, Dean [of the] Division of the Social Sciences

In re: Foreign language requirements for the Ph. D

At a meeting of the faculty of the Division of the Social Sciences held November 18, 1931, the following Statement was adopted as one of the requirements for admission to candidacy for the Ph.D.:

“The ability, demonstrated to the satisfaction of the Board of Examiners, to read two foreign modern languages approved by the department or interdepartmental committee, one of which must be French or German. It is advised that this requirement be met as early as possible in the student’s program of study.”

On January 15, 1943, the faculty of the Division modified the language requirement and made it read as follows:

“Demonstration of the ability to use one or more foreign languages effectively toward the objective of the student’s academic program. This ability in the case of at least one language will be tested by: (I) the passing of an examination administered and evaluated by the board of Examinations; and (II) in addition to the examination, the writing of a paper or series of papers, or oral work, judged satisfactory by the Department in connection with the student’s program of study, in which extensive use is made of works in the foreign language. Departments may at their discretion specify the particular foreign language or languages required of their candidates for the doctorate. Exceptions in individual cases may be made by the Dean of Students on the recommendation of a Department.”

At its meeting on December 4, 1967, the faculty of the Division approved the following motion:

“That each Department or Committee in the Division should have the right to substitute for the divisional language requirement for the Ph.D. degree a requirement that the student demonstrate proficiency in a substantive field other than that of the department in question. The degree of proficiency to be required in such a field should be comparable or superior to that represented by the present divisional language requirement. The department at its option, may require that proficiency be demonstrated in a particular substantive field or may leave to the student, in some or all cases, the option of deciding whether to demonstrate proficiency in a language or a substantive field.”

If a department or committee wishes to act under the motion approved by the faculty of the Division, Mr. Zimring and I urge that it do so with reasonable promptness to permit students to make their plans with full information concerning the requirements. Until a department or committee has taken action and so informs the Office of the Dean it will be assumed that the language requirement as it existed on December 1, 1967, shall continue in force.

At the time a department or committee takes action, if any, to modify its language requirement, it should indicate the date on which any changes become effective and that date could be within a few days after action has been taken. I feel that before a departmental or committee action is made effective that Mr. Zimring should be given time to study it and to determine if his interpretation is the one that has been intended.

Mr. Zimring and I strongly urge that departments and committees accept a self-denying ordinance, namely that changes in the requirements will not be made more than once a year and that changes affecting 1968-69 be made by March 15, 1968.

The action taken with respect to the Divisional language requirement does not change current procedures with respect to examination if a department retains a language requirement. It is my interpretation that while departments can require a supplementary or additional examination to be administered by the department, it cannot dispense with the examinations given by the Office of the University Examiner.

DGJ:bp

Source:  Hoover Institution Archives. Papers of Milton Friedman, Box 194, Folder 4, “Economics Dept A-G”.

_________________________

Linear Algebra
or a Foreign Language

Department of Economics  University of Chicago
May, 1968

To: Graduate Students, Department of Economics
From: Arnold C. Harberger. Chairman

Re: Revision in Foreign Language/ Mathematics Requirement

Foreign Language and Mathematics Requirement

Each Ph.D. candidate, before admission to candidacy for the Ph.D., must demonstrate effective command of relevant mathematical tools, including calculus and matrix algebra. The Department of Economics will accept three courses in calculus (mathematics 151, 152, and 154, or equivalent), and one course in linear algebra (mathematics 250, 252, Business 372, or equivalent) as meeting, respectively, the calculus and matrix algebra requirements.

In place of demonstration of competence in matrix algebra, students may opt to demonstrate proficiency at a high level in a foreign language by means of an examination administered by the Office of the University Examiner and must demonstrate to the satisfaction of the Department ability to translate at sight with reasonable ease material in economics in the foreign language. Any foreign language other than Chinese, French, German, Italian, Japanese, Russian, Spanish, or Swedish must be approved by the Department.

Source:  Hoover Institution Archives. Papers of Milton Friedman, Box 194, Folder 4, “Economics Dept A-G”.

Categories
Exam Questions Harvard Money and Banking

Harvard. Reading list and final exam for Survey of Currency Regulation. Andrew, 1902-1903

Abram Piatt Andrew, Jr. sprang from an assistant professorship of economics at Harvard (following his Ph.D. in 1900) to playing a key staff role in the preparation of the reports of the National Monetary Commission. Ultimately he became a Republican Congressman from Massachusetts, serving from September 1921 until his death in June 1936.

______________________

Official Congressional Biography of
Abram Piatt Andrew, Jr.

ANDREW, Abram Piatt, Jr., a Representative from Massachusetts; born in La Porte, La Porte County, Ind., February 12, 1873; attended the public schools and the Lawrenceville (N.J.) School; was graduated from Princeton College in 1893; member of the Harvard Graduate School of Arts and Sciences 1893-1898; pursued postgraduate studies in the Universities of Halle, Berlin, and Paris; moved to Gloucester, Mass., and was instructor and assistant professor of economics at Harvard University 1900-1909; expert assistant and editor of publications of the National Monetary Commission 1908-1911; director of the Mint 1909 and 1910; Assistant Secretary of the Treasury 1910-1912; served in France continuously for four and a half years during the First World War, first with the French Army and later with the United States Army; commissioned major, United States National Army, in September 1917 and promoted to lieutenant colonel in September 1918; elected as a Republican to the Sixty-seventh Congress to fill the vacancy caused by the resignation of Willfred W. Lufkin; reelected to the Sixty-eighth and to the six succeeding Congresses and served from September 27, 1921, until his death; delegate to the Republican National Conventions in 1924 and 1928; member of the board of trustees of Princeton University 1932-1936; died in Gloucester, Mass., June 3, 1936; remains were cremated and the ashes scattered from an airplane flying over his estate at Eastern Point, Gloucester, Mass.

Source: Abram Piatt Andrew, Jr. entry at the internet Biographical Directory of the United States Congress.

______________________

Economics 8a
Course Description

1902-1903

8a1hf. Money. — A general survey of currency legislation, experience, and theory in recent times. Half-course (first half-year). Mon., Wed., Fri., at 10. Dr. Andrew.

In this course the aim will be to show how the existing monetary systems of the principal countries have come to be, and to analyze the more important currency problems. The course will begin with a brief history of the precious metals, which will be connected, in so far as possible, with the history of prices and the development of monetary theory. The history of coinage legislation in England and Europe and the United States will be traced, and will lead to an extended consideration of the various aspects of the bimetallic controversy.

At convenient points, the experiences of various countries with paper money will also be reviewed, and the influence of such issues upon wages, prices, and trade examined. Some attention, moreover, will be given to the non-monetary means of payment and to the large questions of monetary theory arising from their use.

Systematic reading will be required and will be tested by monthly examinations.

Course 8a1 is open to students who have taken Course 1.

Source: Harvard University. Faculty of Arts and Sciences, Division of History and Political Science [Comprising the Departments of History and Government and Economics], 1902-03. Published in The University Publications, New Series, no. 55. June 14, 1902.

______________________

Economics 8a
Course enrollment
1902-1903

Economics 8a. 1hf. Dr. Andrew. — Money. A general survey of currency legislation, experience, and theory in recent times.

Total 99: 4 Gr., 34 Se., 40 Ju., 14 So., 1 Fr., 6 Others.

Source: Harvard University. Annual Report of the President of Harvard College, 1902-03, p. 68.

______________________

Reading List
Economics 8a
1902-03

BOOKS TO BE PROCURED

Francis A. Walker: International Bimetallism.

J. Laurence Laughlin: History of Bimetallism in the United States.

Leonard Darwin: Bimetallism.

REQUIRED READING

October

Walker: 1-110, 118-183.

Laughlin: 109-206.

Macaulay: History of England, ch. XXI. (Passages concerning the currency and its reform.)

The Bullion Report in Sumner, History of American Currency, Appendix; or in Sound Currency pamphlet, Vol. II, No. 14.
In connection with the Bullion Report consult Macleod: Theory of Credit, 738-760, 551-573; or Theory of Banking, I, 516-539,II, 1-95; or Sumner, American Currency, 231-310.

November

Laughlin: 1-105, 209-280.

Walker: 110-117, 183-9, 217-224.

Darwin: 1-154.

December

Taussig: Recent. Investigations on Prices in the United States, in the Yale Review for November, 1893.

Walker: 190-288.

Darwin: 157-280.

Taussig: The International Silver Situation in the Quarterly Journal of Economics for October, 1896.

January

(To be assigned later.)

Hour examinations will be held Nov. 7 and Dec. 5.

Source: Harvard University Archives. Syllabi, course outlines and reading lists in Economics, 1895-2003. Box 1, Folder “1902-1903.”

Cf. the richer bibliography for this course from the first term 1901-1902.

______________________

ECONOMICS 8a
Mid-year Examination
1902-1903

Arrange answers strictly in the order of the questions
Omit one question

  1. Why did England adopt the gold standard? Why have other countries adopted it?
    State Walker’s and Laughlin’s opinions as well as your own.
  2. In a country with an inconvertible paper currency what are the conditions which control the premium on gold?
    1. according to the Bullion Report.
    2. according to Tooke.
    3. according to your own opinion.

Illustrate from English or American experiences with inconvertible paper.

  1. Trace the general changes in the value of gold in the United States from 1850 to the present time, analyzing so far as possible the reasons for these changes.
  2. Explain what you consider to be the merits and defects in the plans for the Phillipine currency adopted last year by the Senate and the House respectively? What lesson of significance for the Phillipine currency can be drawn from the history of the Latin monetary Union?
  3. Suppose that owing to the increasing gold supply the ratio between gold and silver were to fall, how would Mexican prices and the foreign trade of Mexico tend to be affected?
  4. How would you expect the fall in the value of silver to have affected (a) the trade relations of England and India before 1893? (b) the trade relations of India and China after 1893? How far is theory confirmed by actual experience?
  5. How according to Walker did monetary conditions affect the interests of the American working classes during the last quarter of the 19th century? Explain and criticise the nature of our evidence upon this subject.
    How did Walker and Marshall differ with regard to the effect of monetary conditions upon productive enterprise?
  6. Do falling prices “necessarily enhance the burden of all debts and fixed charges?”
    Illustrate by the experience of the United States during the period from 1873 to 1896, pointing out possible differences between agricultural and mercantile debts.
  7. Under what circumstances might a higher level of prices be maintained by means of book credits? Under what circumstances might a continuous rise in prices be effected by means of such credits?

Source: Harvard University Archives. Mid-year Examinations 1852-1943. Box 6. Papers (in the bound volume Examination Papers Mid-years 1902-1903).
Also included in: Harvard University Archives. Examination Papers 1873-1915. Box 6. Papers Set for Final Examinations in History, Government, Economics, History of Religions, Philosophy, Education, Fine Arts, Architecture, Landscape Architecture, Music in Harvard College, June 1903 (in the bound volume Examination Papers 1902-1903).

Image Source: 1911 portrait of Abram Piatt Andrew, Jr. by Anders Born at Boston’s Isabella Stewart Gardner Museum. Wikimedia Commons.

 

Categories
Exam Questions Harvard Industrial Organization Labor

Harvard. Problems of labor and industrial organization, Student Reports, Exams. Ripley, 1902-1903

 

William Zebina Ripley began his career as racial social anthropologist, with a scientific legacy that could have reduced him to the fate of a poster-child in later histories of popular and academic racism. At the latest, his work as transportation expert for the United States Industrial Commission in 1900 helped to divert his attention to disputes between organized labor and organized capital and away from “problems of immigration”. And so we find his course “Problems of Labor and Industrial Organization” at Harvard which is the subject of this post. Besides a thick course description in the division announcement of its course offerings for 1902-03, course enrollment figures, and the semester final exams, the Harvard archives also has a copy of a four-page printed instructions for the student reports to be prepared during the first semester covering “problems of labor”.

______________________

Economics 9
Course Description
1902-1903

  1. Problems of Labor and Industrial Organization. , Th., and (at the pleasure of the instructor) Sat., at 10. Professor Ripley.

The work of this course will consist of two parts, one concerned mainly with the economic and social questions relating to labor, with especial reference to legislation; the other with the fiscal and industrial organization of capital, especially in the corporate form.

Among the topics included under the first head will be the following, viz.: methods of remuneration, profit sharing, cooperation, sliding scales, and collective bargaining; labor organizations; factory legislation in all its phases in the United States and Europe; strikes, strike legislation and judicial interpretation, conciliation and arbitration; employers’ liability and compulsory compensation acts; compulsory insurance with particular reference to European experience; provident institutions, friendly societies, building and loan associations; the problem of the unemployed; apprenticeship, and trade and technical education.

With reference to the organization of capital and especially the economics of corporations, the principal topics will be industrial combination and the so-called trust problem. This will be considered in all its phases, with comparative study of the conditions in the United States and European countries. The growth and development of corporate enterprise, promotion, capitalization and financing, publicity of accounting, the liability of directors and underwriters, will be illustrated fully by the study of cases, not from their legal but from their purely economic aspects; and the effects of industrial combination and integration upon efficiency, profits, wages, the rights of investors, prices, industrial stability, the development of export trade, and international competition will be considered in turn.

The course is open to students who have taken Economics 1. Systematic reading and report work will be assigned from time to time.

Source: Harvard University. Faculty of Arts and Sciences, Division of History and Political Science[Comprising the Departments of History and Government and Economics], 1902-03. Published in The University Publications, New Series, no. 55. June 14, 1902.

______________________

Economics 9
Enrollment
1902-1903

Economics 9. Professor Ripley. — Problems of Labor and Industrial Organization.

Total 112: 3 Gr., 26 Se., 55 Ju., 17 So., 11 Others.

Source: Harvard University. Annual Report of the President of Harvard College, 1902-03, p. 68.

______________________

ECONOMICS 9
ASSIGNMENT OF REPORTS

GROUP A

Students will report upon the comparative conditions respecting Trade Union organization, functions, and efficiency in corresponding industries in the United States and Great Britain. The particular, industry assigned to each man is indicated by a number on the enrolment slip, which refers to the Trade Union number on the appended list of National Labor Organizations.

GROUP B

Students will report upon the comparative efficiency of Trade Union organization in two distinct lines of industry in the United States. Numbers against the names on the enrolment slip refer to the numbered Trade Union list, appended hereto.

GROUP C

Students will report upon the nature of Trade Union organization in two distinct lines of industry in Great Britain. Names on the enrolment slip as numbered refer to the industries concerned in the appended list of Trade Unions.

→ The letters preceding the assignment number against the student’s name refer to the group in which the report is to be made. Thus, for example: “8A” on the enrolment slip indicates that the student is to report upon the Cotton Spinners’ Unions in the United States and Great Britain; “1 & 8B,” that a comparison of the Spinners’ and of the Boot and Shoe Workers’ Organizations in the United States is expected : while “1 & 8C” calls for the same comparison for the two industries in Great Britain.

NATIONAL LABOR ORGANIZATIONS IN THE UNITED STATES

A star indicates that the Trade Union journal is in the Library. [Loeb Fund.]

*The Knights of Labor
*The American Federation of Labor

  1. Boot and Shoe Workers’ Union.
  2. The United Hatters of North America.
  3. The United Garment Workers of America.
  4. The Journeymen Tailors’ Union of America.
  5. Custom Clothing Makers’ Union of America.
  6. International Ladies’ Garment Workers’ Union.
  7. The Shirt, Waist and Laundry Workers’ International Union.
  8. National Spinners’ Association of America.
  9. The Elastic Goring Weavers’ Amalgamated Association of the United States of America.
  10. International Union of Textile Workers.
  11. Trunk and Bag Workers’ International Union of America.
  12. *International Typographical Union of North America.
  13. German-American Typographia.
  14. International Printing Pressmen and Assistants’ Union of North America.
  15. International Brotherhood of Bookbinders.
  16. Lithographers’ International Protective and Beneficial Association.
  17. International Steel and Copperplate Printers’ Union of the United States of America.
  18. Bricklayers and Masons’ International Union of America.
  19. *United Brotherhood of Carpenters and Joiners of America.
  20. Amalgamated Society of Carpenters and Joiners.
  21. International Brotherhood of Electrical Workers.
  22. *Granite Cutters’ National Union of the United States of America.
  23. Brotherhood of Painters, Decorators, and Paper Hangers of America.
  24. Operative Plasterers’ International Association.
  25. United Association of Journeymen Plumbers, Gas Fitters, Steam Fitters, and Steam Fitters’ Helpers.
  26. National Association of Steam and Hot-Water Fitters and Helpers.
  27. Journeymen Stone Cutters’ Association of North America.
  28. Mosaic and Encaustic Tile Layers and Helpers’ International Union.
  29. Glass Bottle Blowers’ Association.
  30. American Flint Glassworkers’ Union.
  31. *Amalgamated Glassworkers’ International Association.
  32. National Brotherhood of Operative Potters.
  33. *United Mine Workers of America.
  34. Northern Mineral Mine Workers’ Progressive Union.
  35. Amalgamated Woodworkers’ International Union.
  36. United Order of Box Makers and Sawyers.
  37. *Piano and Organ Workers’ International Union.
  38. International Wood Carvers’ Association.
  39. Coopers’ International Union.
  40. Carriage and Wagon Workers’ International Union.
  41. National Amalgamated Association of Iron, Steel, and Tin Workers.
  42. *International Association of Machinists.
  43. Amalgamated Society of Engineers.
  44. *Brotherhood of Boiler Makers and Iron Ship Builders.
  45. International Association of Allied Metal Mechanics.
  46. Metal Polishers, Buffers, Platers, and Brass Workers’ International Union.
  47. Amalgamated Sheet Metal Workers’ International Association.
  48. *Iron Molders’ Union.
  49. Pattern Makers’ League.
  50. Core Makers’ International Union.
  51. Grand Union of the International Brotherhood of Blacksmiths.
  52. Chain Makers’ National Union.
  53. Stove Mounters and Steel Range Workers’ International Union.
  54. Tin Plate Workers’ International Protective Association.
  55. American Wire Weavers’ Protective Association.
  56. Metal Trades’ Federation of North America.
  57. *International Seamen’s Union.
  58. National Marine Engineers’ Beneficial Association.
  59. International Longshoremen’s Association.
  60. Amalgamated Association of Street Railway Employees.
  61. Switchmen’s Union.
  62. Journeymen Bakers’ and Confectioners’ International Union.
  63. Journeymen Barbers’ International Union.
  64. National Union of the United Brewery Workmen.
  65. *National Brickmaker’s Alliance.
  66. International Broom Makers’ Union.
  67. *Cigar Makers’ International Union.
  68. Retail Clerks’ International Protective Association.
  69. Team Drivers’ International Union.
  70. International Union of Steam Engineers.
  71. National Brotherhood of Coal Hoisting Engineers.
  72. Watch Case Engravers’ International Association.
  73. International Brotherhood of Stationary Firemen.
  74. International Union of Journeymen Horseshoers.
  75. Hotel and Restaurant Employees’ International Alliance and Bartenders’ International League.
  76. International Jewelry Workers.
  77. The United Brotherhood of Leather Workers on Horse Goods.
  78. National Association of Letter Carriers.
  79. *Amalgamated Meat Cutters and Butcher Workmen.
  80. American Federation of Musicians.
  81. International Brotherhood of Oil and Gas Well Workers.
  82. United Brotherhood of Paper Makers.
  83. National Alliance of Theatrical Stage Employees.
  84. National Stogie Makers’ League.
  85. *Tobacco Workers’ International League.
  86. Upholsterers’ International Union.
  87. *Brotherhood of Locomotive Engineers.
  88. *Order of Railway Conductors of America.
  89. *Brotherhood of Locomotive Firemen.
  90. Brotherhood of Railroad Trainmen.
  91. *Order of Railroad Telegraphers.
  92. Brotherhood of Railway Truckmen.
  93. Switchmen’s Union of North America.

The constitutions of most of the Trades Unions for the United States will be found in Vol. XVII, Reports, U. S. Industrial Commission. Similar data for Great Britain is in the Appendix to “Foreign Reports, Vols. 1-2,”Royal Commission on Labour, pp. 15-324. [Volume I, United States; Volume II, Colonies and Indian Empire] [Both reserved in Gore Hall.] Additional evidence as to labor conditions in each industry will be found in Vols. VII, VIII, XII, XIV, and XVII, U. S. Industrial Commission (consult Digest and Index in each volume); and in the Reports of the British Royal Commission. The student should also consult Charles Booth’s Life and Labor of the People;

[(Original) Volume I, East London; (Original) Volume II, London; (Original) Appendix to Volume II; Note: the previous three original volumes were re-printed as four volumes that then were followed by Volume V, Population Classified by Trades; Volume VI, Population Classified by Trades (cont.); Volume VII, Population Classified by Trades; Volume VIII, Population Classified by Trades (cont.); Volume IX, Comparisons, Survey and Conclusions];

Webbs, Industrial Democracy; and other books reserved in Gore Hall.

Data respecting the various unions among railroad employees in the United States will be found in a separate section on Railway Labor, in Vol. XVII, U. S. Industrial Commission: as also in Vols. IV and IX. (See Digests and Indexes.)

In cases where the American Trade Union journal is not in the library, the student will be expected to procure at least one copy from the Secretary of the Union. [See list of post office addresses posted with the enrolment slip.] These are to be filed with the report.

→ Exact references by title, volume and page must be given in foot notes for all facts cited. This condition is absolutely imperative. Failure to comply with it will vitiate the entire report.

Source: Harvard University Archives. Syllabi, course outlines and reading lists in Economics, 1895-2003. Box 1, Folder “Economics, 1902-1903”.

______________________

Economics 9
Mid-Year Examination
1902-1903

  1. What is the “fellow-servant doctrine” as applied to the interpretation of Employers’ Liability? Upon what reasons was it based; wherein did it fail; and how was it remedied?
  2. Describe, briefly, the causes and results of five great strikes in the United States since 1870.
  3. What was the recent Taff Vale case in Great Britain, and why was it so important?
  4. What is the position of the leading political parties and statesmen in England on labor questions?
  5. What is the general attitude of the American courts toward—
    1. The rights of strikers?
    2. Boycotts?
    3. Non-union men?
  6. What is an injunction? What are its merits and evils?
  7. Where has compulsory arbitration of labor disputes been attempted? What is the main objection to such a policy?
  8. What is the relative importance of the different causes of labor disputes? Are they different in different countries?

Source: Harvard University Archives. Mid-year Examinations 1852-1943. Box 6. Papers (in the bound volume Examination Papers Mid-years 1902-1903).

Economics 9b
Year-end examination
1902-1903

  1. Outline with approximate dates, the development of the so-called Trust Problem in the United States.
  2. Show how the methods of promotion usually adopted have affected capitalization; illustrating by a case familiar to you.
  3. In what ways does the financing of industrials differ from railways; and what has been the principal effect upon their dividend policy?
  4. What are Jenks’ main conclusions as to the effect of combination upon prices? How do these compare with Ely’s?
  5. What are the three main inducements for transforming a partnership into a corporation?
  6. Contrast the attitude of the English common law toward monopoly with that of the law in Germany and France.
  7. What has been the attitude of the U.S. Steel Corporation toward its employees? Illustrate by concrete experience.
  8. Is “publicity” a sufficient remedy for the existing abuses of corporate management, in your opinion? Give specific reasons for your view.
  9. What was the decision in the Northern Securities Co. case, and what may be its possible effect?

Source: Harvard University Archives. Examination Papers 1873-1915. Box 6. Papers Set for Final Examinations in History, Government, Economics, History of Religions, Philosophy, Education, Fine Arts, Architecture, Landscape Architecture, Music in Harvard College, June 1903 (in the bound volume Examination Papers 1902-1903).

Image Source: Harvard University Archives.  William Zebina Ripley [photographic portrait, ca. 1910], J. E. Purdy & Co., J. E. P. & C. (1910). Colorized by Economics in the Rear-view Mirror.

Categories
Funny Business Theory

Funny business. Emily Dickinson explains the 2nd theorem of welfare economics. 2023

I just asked ChatGPT

« Explain the second theorem of welfare economics in the style of an Emily Dickinson death poem. »

[Spoiler alert: her death poems can be sung to the tune of The Yellow Rose of Texas]

The response:

The Second Theorem lies in wait,
Its logic cold and clear,
It whispers to the market’s fate,
And marks the end so near.

With mathematics sharp as a knife,
It cuts through trade and choice,
Revealing truths about our life,
In market’s still, small voice.

It states that perfect competition,
Is what leads to best result,
For welfare of each citizen,
And market’s every pulse.

Yet still the Theorem’s voice is kind,
For it is but a guide,
To lead us to a better mind,
And market where all thrive.

So heed the Second Theorem’s call,
And let its wisdom lead,
For perfect competition’s all,
That our market needs.

Image Source: Wikipedia.deDaguerreotype of Emily Dickinson, c. early 1847. It is presently located in Amherst College Archives & Special Collections. Colorized by Economics in the Rear-view Mirror.